Đến nội dung

lahantaithe99 nội dung

Có 878 mục bởi lahantaithe99 (Tìm giới hạn từ 05-06-2020)



Sắp theo                Sắp xếp  

#483339 Topic về Bất đẳng thức, cực trị THCS

Đã gửi bởi lahantaithe99 on 15-02-2014 - 22:28 trong Bất đẳng thức và cực trị

1)$Cho$$x+y+xy$$=24$. Tìm GTNN$x^{2}+y^{2}$

2)$Cho$$x^2+y^2-xy=4$. TÌm GTLN và GTNN của$x^2+y^2$

1.

$x+y+xy=24\leq x+y+\frac{(x+y)^2}{4}\Rightarrow x+y\geq 8$

$\Rightarrow x^2+y^2\geq \frac{(x+y)^2}{2}\geq \frac{8^2}{2}=32$

2.

$x^2+y^2-xy=4\geq 2xy-xy=xy$

$\Rightarrow x^2+y^2=4+xy\leq 4+4=8$




#486960 Topic về Bất đẳng thức, cực trị THCS

Đã gửi bởi lahantaithe99 on 15-03-2014 - 17:07 trong Bất đẳng thức và cực trị

Cho các số dương a, b, c thỏa mãn a+ b2 + c2 =3

CMR: $\frac{a}{\sqrt{b}}$ + $\frac{b}{\sqrt{c}}$ + $\frac{c}{\sqrt{a}}$  $\geq$  a + b + c

Bài này bạn đã đăng [r bên này rồi mà!

Cách khác

http://diendantoanho...s-cao-xuân-huy/




#486926 Topic tổng hợp các bài toán về phương trình nghiệm nguyên.

Đã gửi bởi lahantaithe99 on 15-03-2014 - 09:42 trong Số học

216.Cho $\overline{abc}=p$ (p là số nguyên tố). Chứng minh pt \[ax{}^{2}+b\text{x}+c=0\] vô nghiệm

Bài 216: Hình như đề bài thiếu: Phải là CM phương trình đã cho vô nghiệm hữu tỉ (nghiệm nguyên)

Giả sử pt đã cho có nghiệm hữu tỉ

Ta có $\Delta _{x}=b^2-4ac$ $(1)$

Từ $\overline{abc}=p\Leftrightarrow c=p-100a-10b$

$\Leftrightarrow 4ac=4ap-400a^2-40ba$

Thay vào $(1)$ : $\Delta =b^2-4ap+400a^2+40ab=(20a+b)^2-4ap$

Để pt có ngiệm hữu tỉ thì $\Delta =(20a+b)^2-4ap=k^2 (k\in \mathbb{Z})$

$\Leftrightarrow (20a+b-k)(20+b+k)=4ap\vdots p$

suy ra $20a+b+k\vdots p$ hoặc $20a+b-k\vdots p$

Mà $20a+b-k;20a+b+k\neq 0$ $\Rightarrow 20a+b+k;20a+b-k\geqslant p$

(điều này vô lý vì $\overline{abc}=p$

Vậy điều giả sử là vô lý

Do đó ta có đpcm




#505750 Bất đẳng thức chuẩn bị cho kì thi THPTQG 2015-2016

Đã gửi bởi lahantaithe99 on 11-06-2014 - 13:06 trong Bất đẳng thức và cực trị

Tìm giá trị lớn nhất của biểu thức

$$F=\dfrac{a}{\sqrt{a^2+b+c}}+\dfrac{b}{\sqrt{b^2+c+a}}+\dfrac{c}{\sqrt{c^2+c+a}}$$

Trong đó $a,b,c$ là các số thực dương thỏa mãn$a^2+b^2+c^2=3$

Áp dụng Bunhia

 

$(a^2+b+c)(1+b+c)\geqslant (a+b+c)^2\Rightarrow \sqrt{a^2+b+c}\geq \frac{a+b+c}{\sqrt{1+b+c}}$

 

Suy ra $F\leqslant \frac{\sum a\sqrt{1+b+c}}{a+b+c}\leqslant \frac{\sqrt{(a+b+c)(a+b+c+2ab+2bc+2ca)}}{a+b+c}=\frac{\sqrt{3(3+2a+2b+2c)}}{a+b+c}$

 

$\Leftrightarrow F\leqslant \sqrt{\frac{a+b+c+2ab+2bc+2ac}{a+b+c}}$

 

Lại có $2ab+2bc+2ac\leqslant \frac{2(a+b+c)^2}{3}\Rightarrow F\leqslant \sqrt{\frac{3+2(a+b+c)}{3}}\leqslant \sqrt{3}$




#476279 Bất đẳng thức chuẩn bị cho kì thi THPTQG 2015-2016

Đã gửi bởi lahantaithe99 on 09-01-2014 - 12:32 trong Bất đẳng thức và cực trị

Cho x,y,z là số thực dương. Chứng minh

$\frac{5x^3-y^3}{xy+3x^2}+\frac{5y^3-z^3}{zy+3y^2}+\frac{5z^3-x^3}{zx+3z^2}\leq x+y+z$

Áp dụng công thức $x^3+y^3\geq xy(x+y)\Rightarrow y^3\geq xy(x+y)-x^3$

Do đó $\frac{5x^3-y^3}{xy+3x^2}\leq \frac{6x^3-xy(x+y)}{xy+3x^2}=\frac{6x^2-xy-y^2}{y+3x}=2x-y$

Làm tương tự như vậy đối với các phân thức còn lại ta được

$\frac{5x^3-y^3}{xy+3x^2}+\frac{5y^3-z^3}{zy+3y^2}+\frac{5z^3-x^3}{zx+3z^2}\leq 2x-y+2y-z+2z-x=x+y+z$ (đpcm)

Dấu = xảy ra khi $x=y=z$




#495286 Bất đẳng thức chuẩn bị cho kì thi THPTQG 2015-2016

Đã gửi bởi lahantaithe99 on 26-04-2014 - 19:07 trong Bất đẳng thức và cực trị

Cho ba số thực dương x.y.z. Tìm GTNN của biểu thức:

P = $\frac{x}{y+z}+\frac{y}{z+x}+\frac{z}{x+y}+\frac{y+z}{x}+\frac{z+x}{y}+\frac{x+y}{z}$

Ta có

 

$\frac{x}{y+z}+\frac{y}{x+z}+\frac{z}{x+y}\geqslant \frac{3}{2}$ (BĐT Nesbit) $(1)$

 

$\frac{y+z}{x}+\frac{z+x}{y}+\frac{x+y}{z}\geqslant 3\sqrt[3]{\frac{(x+y)(y+z)(z+x)}{xuz}}\geqslant 3\sqrt[3]{8}=6$ $(2)$

 

(áp dụng BĐT $AM-GM$)

 

Từ $(1);(2)\Rightarrow VT\geqslant \frac{15}{2}$




#494364 $\boxed{\text{Chuyên Đề}}$ Bất đẳng thức - Cực trị

Đã gửi bởi lahantaithe99 on 21-04-2014 - 17:35 trong Bất đẳng thức và cực trị

Đóng góp 2 bài vậy !! :P

Bài 150 : Cho $a;b;c$ là các số thực dương. Tìm $GTNN$ của : 
$$Q=\sum \sqrt[4]{\frac{a}{b+c}}+\sum \sqrt{\frac{b+c}{a}}$$
 

Bài này nhìn số má trâu bò quá!  :wacko:  :wacko:  :wacko:

$150/$

Áp dụng BĐT Cauchy ta có

 

$Q\geqslant 3\left [ \sqrt[12]{\frac{abc}{(a+b)(b+c)(c+a)}} +\sqrt[6]{\frac{(a+b)(b+c)(c+a)}{abc}}\right ]$

 

$=3\left [ \sqrt[12]{\frac{abc}{(a+b)(b+c)(c+a)}}+\frac{1}{\sqrt[4]{8}}.\sqrt[6]{\frac{(a+b)(b+c)(c+a)}{abc}} \right ]+3(1-\frac{1}{\sqrt[4]{8}})\sqrt[6]{\frac{(a+b)(b+c)(c+a)}{abc}}$

 

 Cô si cho biểu thức thứ nhất

 

Biếu thức $(1)$ $\geqslant 2\sqrt[24]{\frac{(a+b)(b+c)(c+a)}{512abc}}\geqslant 2.\sqrt[24]{\frac{1}{64}}=2.\sqrt[4]{\frac{1}{2}}$

 

Biểu thức $(2)$ $(1-\frac{1}{\sqrt[4]{8}})\sqrt[6]{\frac{(a+b)(b+c)(c+a)}{abc}}\geqslant (1-\frac{1}{\sqrt[4]{8}})\sqrt[6]{8}=\sqrt{2}-\sqrt[4]{\frac{1}{2}}$

 

Cộng vế suy ra $Q\geqslant 3(\sqrt[4]{\frac{1}{2}}+\sqrt{2})$

 

P/s: bài này cồng kềnh tốn bao nhiêu t/g, mà lại còn k biết có đúng k nữa 




#490839 $\boxed{\text{Chuyên Đề}}$ Bất đẳng thức - Cực trị

Đã gửi bởi lahantaithe99 on 05-04-2014 - 19:12 trong Bất đẳng thức và cực trị

Bài 148. Cho $a,b,c>0.$ Tìm $maxP=\frac{1}{\sqrt{a^2+b^2+c^2}}-\frac{2}{(a+1)(b+1)(c+1)}$

Bài này theo mình không tìm đc max (ý kiến cá nhân) 

Thấy tính đối xứng của $a,b,c$ và $a,b,c>0$ nên chác dấu $=$ xảy ra khi $a=b=c$

Thay $a=b=c$ bằng một số giá trị thì cứ giá trị càng nhỏ thì $P$ càng lớn, Vd như $a=b=c=0,0001$ thì $P$ lên đến hơn $5000$, vậy nếu $a=b=c$ bàng một số nhỏ hơn nữa thì không biết giá trị là bao nhiêu???????????




#495522 $\boxed{\text{Chuyên Đề}}$ Bất đẳng thức - Cực trị

Đã gửi bởi lahantaithe99 on 27-04-2014 - 19:00 trong Bất đẳng thức và cực trị

 

 

Cách khác đây

152)
Đặt $x = \sqrt{a} ; y = \sqrt{b} ; z = \sqrt{c} $ khi đó phải chứng minh
$$\frac{1}{x^{2} + y^{2} + 1} + \frac{1}{y^{2} + z^{2} + 1} + \frac{1}{x^{2} + z^{2} + 1} \leq 1 $$
 

 

Mình thấy chứng minh bài này dựa vào số trung gian là $1$ thì không được, vì $\sum \frac{1}{a+2}$ không phải luôn $\geqslant 1$

nếu $a=\frac{1}{3};b=3;c=1$

Chưa xem hết bài của Hoàng nhưng mình thấy khá rối và $x=\sqrt{a};y=\sqrt{b};z=\sqrt{c}$ thì suy ra luôn $xyz=1$ tại sao lại xét $xyz\geqslant 1$




#511477 $\boxed{\text{Chuyên Đề}}$ Bất đẳng thức - Cực trị

Đã gửi bởi lahantaithe99 on 07-07-2014 - 16:35 trong Bất đẳng thức và cực trị

Bài 191:Cho a,b,c là các số thực.CMR: $\left ( \prod (a+b-c) \right )^{2}\geq \prod (a^{2}+b^{2}-c^{2}).$

Bạn tham khảo ở đây nhé

http://diendantoanho...b2c2-a2c2a2-b2/




#510810 $\boxed{\text{Chuyên Đề}}$ Bất đẳng thức - Cực trị

Đã gửi bởi lahantaithe99 on 04-07-2014 - 20:03 trong Bất đẳng thức và cực trị

 

Bài 154: Cho $a,b,c>0$. CMR:

$(1+a+b+c)(1+ab+bc+ca)\geq 4\sqrt{2(a+bc)(b+ca)(c+ab)}$

 

Bài 154.

 

Áp dụng BĐT $AM-GM$ ta có

 

$(a+bc)(b+ac)\leqslant \frac{(a+bc+b+ac)^2}{4}=\frac{(a+b)^2(c+1)^2}{4}$

 

Suy ra $4\sqrt{2(a+bc)(b+ac)(c+ab)}\leqslant 2\sqrt{(a+b)(b+c)(c+a)(a+1)(b+1)(c+1)}$ $(1)$

 

Lại có 

 

$(1+a+b+c)(1+ab+bc+ac)=1+a+b+c+ab+bc+ac+(ab+bc+ac)(a+b+c)$

 

$=1+a+b+c+ab+bc+ac+abc+(a+b)(b+c)(c+a)$

 

$=(a+1)(b+1)(c+1)+(a+b)(b+c)(c+a)\geqslant 2\sqrt{(a+1)(b+1)(c+1)(a+b)(b+c)(c+a)}$

 

($AM-GM$) $(2)$

 

Từ $(1);(2)$ ta có đpcm




#506586 $\boxed{\text{Chuyên Đề}}$ Bất đẳng thức - Cực trị

Đã gửi bởi lahantaithe99 on 14-06-2014 - 13:54 trong Bất đẳng thức và cực trị

178) Cho $x,y,z>1$ sao cho $\sum \frac{1}{x}=2$. C/m 
$\sqrt{x+z+y} \ge \sum \sqrt{x-1}$ ( Iran 1998) 
179) C/m với $a,b,c>0$ thì : 
$\sum \frac{(b+c-a)^2}{a^2+(b+c)^2} \ge \frac{3}{5}$ ( Nhật Bản 1997)

178

 

Áp dụng BĐT Bunhiacopxki

 

$(\sum \sqrt{x-1})^2\leqslant (x+y+z)(\frac{x-1}{x}+\frac{y-1}{y}+\frac{z-1}{z})=(x+y+z)(3-\sum \frac{1}{x})=x+y+z$

 

$\Rightarrow \sqrt{x+y+z}\geqslant \sum \sqrt{x-1}$

 

179

 

Không mất tính tổng quát ta chuẩn hóa $a+b+c=3$

 

Khi đó cần chứng minh $\sum \frac{(3-2a)^2}{a^2+(3-a)^2}\geqslant \frac{3}{5}$

 

Ta đi chứng minh BĐT sau đúng $\frac{(3-2a)^2}{a^2+(3-a)^2}\geqslant \frac{23}{25}-\frac{18a}{25}$

 

$\Leftrightarrow (a-1)^2(a+\frac{1}{2})\geqslant 0$ (luôn đúng với mọi số dương $a$)

 

Thiết lập các BĐT tương tự rồi cộng lại ta đpcm




#489815 $\boxed{\text{Chuyên Đề}}$ Bất đẳng thức - Cực trị

Đã gửi bởi lahantaithe99 on 31-03-2014 - 17:43 trong Bất đẳng thức và cực trị

Theo hệ thức Viet thì $x_{1}.x_{2}=1\Rightarrow x_{1}=\frac{1}{x_{2}}$

 

Áp dụng BĐT Cô si thì 

 

$T=x_{1}^4+x_{2}^2=\frac{1}{x_{2}^4}+x_{2}^2=\frac{1}{x_{2}^4}+\frac{x_{2}^2}{2}+\frac{x_{2}^2}{2}\geqslant 3.\sqrt[3]{\frac{1}{4}}$




#495498 $\boxed{\text{Chuyên Đề}}$ Bất đẳng thức - Cực trị

Đã gửi bởi lahantaithe99 on 27-04-2014 - 17:51 trong Bất đẳng thức và cực trị

 

Tương tự : 

$\frac{1}{b+c+1} \le \dfrac{\sqrt[3]{a}}{\sqrt[3]{a}+\sqrt[3]{b}+\sqrt[3]{c} }$

$\frac{1}{c+a+1} \le \dfrac{\sqrt[3]{b}}{\sqrt[3]{a}+\sqrt[3]{b}+\sqrt[3]{c} }$

$\Rightarrow \frac{1}{a+b+1} +\frac{1}{b+c+1}+\frac{1}{c+a+1}\leq 1$

Ta cần chứng minh $\frac{1}{2+a}+\frac{1}{2+b}+\frac{1}{2+c}\geq 1$

Thật vậy, áp dụng bdt $Cauchy$, ta có :

$\frac{1}{2+a}+\frac{2+a}{9}\geq \frac{2}{3}$

$\frac{1}{2+b}+\frac{2+b}{9}\geq \frac{2}{3}$

$\frac{1}{2+c}+\frac{2+c}{9}\geq \frac{2}{3}$

$\Rightarrow \frac{1}{2+a}+ \frac{1}{2+b}+ \frac{1}{2+c}\geq 2-\frac{6+a+b+c}{9}$
                                                     $  \geq 2-\frac{6+3\sqrt[3]{abc}}{9}\geq 1$
$\Rightarrow dpcm$
Dấu ''$=$'' xảy ra $\Leftrightarrow  a=b=c=1$
 
p.s: bạn xem có sai chỗ nào không ? 

 

Ngược dấu rồi anh ơi




#488635 $\boxed{\text{Chuyên Đề}}$ Bất đẳng thức - Cực trị

Đã gửi bởi lahantaithe99 on 24-03-2014 - 22:44 trong Bất đẳng thức và cực trị

chỗ đó hình như sai x2+y2     đâu?

Sai ở đâu ??????????????

Chẳng hiểu, em có thể chỉ rõ ra k

$x^2+y^2+2xy=(x+y)^2$ bình phương thêm lần nữa là mũ $4$




#484668 $\boxed{\text{Chuyên Đề}}$ Bất đẳng thức - Cực trị

Đã gửi bởi lahantaithe99 on 24-02-2014 - 20:44 trong Bất đẳng thức và cực trị

 

cho các số thực x y z thỏa mãn xy+yz+xz=1
tìm min của
p=x^{2}+y^{2}+2z^{2}
:icon10:  :icon10:  :icon10:  :icon10:  :icon10:  :icon10:

 

Phiền bạn lần sau gõ Latex cho dễ nhìn.

Áp dụng bđt Cô si

Ta có $\frac{\sqrt{5}-1}{2}x^2+\frac{\sqrt{5}-1}{2}y^2\geq (\sqrt{5}-1)xy$

$\frac{3-\sqrt{5}}{2}x^2+z^2\geq (\sqrt{5}-1)xz$

$\frac{3-\sqrt{5}}{2}y^2+z^2\geq (\sqrt{5}-1)yz$

Cộng theo từng vế ta có $x^2+y^2+2z^2\geq (\sqrt{5}-1)(xy+yz+zx)=\sqrt{5}-1$




#482742 $\boxed{\text{Chuyên Đề}}$ Bất đẳng thức - Cực trị

Đã gửi bởi lahantaithe99 on 12-02-2014 - 17:34 trong Bất đẳng thức và cực trị

Daicagiangho1998 nhớ trích dẫn đề nha.

 

47) Cho $a;b;c>0$ thỏa $abc=2$. Cmr: $\sum a^3\geq \sum a\sqrt{b+c}$

 

 

Ta có $2(a^3+b^3+c^3)\geq ab(a+b)+bc(b+c)+ac(a+c)=a^2(b+c)+b^2(c+a)+c^2(a+b)$

$\geq \frac{(\sum a\sqrt{b+c})^2}{3}\Rightarrow 6(a^3+b^3+c^3)\geq (\sum a\sqrt{b+c})^2$ $(1)$

Mà $a^3+b^3+c^3\geq 3abc=6\Rightarrow (a^3+b^3+c^3)^2\geq 6(a^3+b^3+c^3)$ $(2)$

$(1);(2)\Rightarrow \sum a^3\geq \sum a\sqrt{b+c}$




#482709 $\boxed{\text{Chuyên Đề}}$ Bất đẳng thức - Cực trị

Đã gửi bởi lahantaithe99 on 12-02-2014 - 13:11 trong Bất đẳng thức và cực trị

 

 

40) Cho $a;b;c>0$ thỏa $a^2+b^2+c^2=1$. Cmr: $\sum \frac{a}{b^2+c^2}\geq \frac{3\sqrt{3}}{2}$

$\sum \frac{a}{b^2+c^2}=\sum \frac{a(a^2+b^2+c^2)}{b^2+c^2}=\sum a+\sum \frac{a^3}{b^2+c^2}$

Bằng $AM-GM$ dễ có $\left\{\begin{matrix} (a^2+b^2+c^2)(a+b+c)=a+b+c\geq 3\sum a^2b & \\ (a^2+b^2+c^2)(a+b+c)=a+b+c\geq 3\sum ab^2& \end{matrix}\right.$

$\Rightarrow \sum a\geq \frac{\sum a^2b+\sum ab^2}{2}$

$\sum \frac{a^3}{b^2+c^2}=\sum \frac{a^4}{ab^2+ac^2}\geq \frac{(a^2+b^2+c^2)^2}{\sum ab^2+\sum a^2b}$

$\Rightarrow VT\geq \frac{\sum ab^2+\sum a^2b}{2}+\frac{1}{\sum a^2b+\sum ab^2}$

$\geq\frac{3\sqrt{3}}{2}$




#482617 $\boxed{\text{Chuyên Đề}}$ Bất đẳng thức - Cực trị

Đã gửi bởi lahantaithe99 on 11-02-2014 - 21:48 trong Bất đẳng thức và cực trị

 

31) Cho $\left\{\begin{matrix}a_1;a_2;...;a_n>0 & & \\ a_1+a_2+...+a_n=1 & & \end{matrix}\right.$. Cmr: $(\frac{1}{a_1}-1)(\frac{1}{a_2}-1)...(\frac{1}{a_n}-1)\geq (n-1)^n$

 

 

 

Ta có $a_{1}+...+a_{n}=1\Rightarrow 1-a_{1}=a_{2}+...+a_{n}\geq (n-1).\sqrt[n-1]{a_{2}...a_{n}}$
Tươn tự $1-a_{n}\geq (n-1).\sqrt[n-1]{a_{1}...a_{n-1}}$
$\Rightarrow \prod (1-a_{1})\geq (n-1)^2.a_{1}...a_{n}$
$\Rightarrow \prod (\frac{1}{a_{1}}-1)\geq (n-1)^2$



#486039 $\boxed{\text{Chuyên Đề}}$ Bất đẳng thức - Cực trị

Đã gửi bởi lahantaithe99 on 06-03-2014 - 18:44 trong Bất đẳng thức và cực trị

107,  Cho các số thực dương x, y, z thoả mãn $\frac{1}{xy}$   +   $\frac{1}{yz}$   $\frac{1}{zx}$  >   0

Tìm GTNN của biểu thức S  =  $\frac{x^{2}}{yz}$    +    $\frac{y^{2}}{zx}$    +    $\frac{z^{2}}{xy}$

Không biết có phải mk nhầm ko chứ mk nghĩ đk $\sum \frac{1}{xy}>0$ chả cần thiết >:)

Theo bđt S.Vac

$S\geq \frac{(x+y+z)^2}{xy+yz+xz}\geq 3$

(do có bđt quen thuộc là $(x+y+z)^2\geq 3(xy+yz+zx)$




#486315 $\boxed{\text{Chuyên Đề}}$ Bất đẳng thức - Cực trị

Đã gửi bởi lahantaithe99 on 08-03-2014 - 21:05 trong Bất đẳng thức và cực trị

sai chỗ màu đỏ này:$b\leq ...;b\leq ...$

Chỗ màu xanh : $b\leq ...$ nhưng chắc gì $a<\frac{3}{2}\Rightarrow b<\frac{3}{2}$

Cậu ko đọc kỹ đề bài sao. ĐỀ BÀI CHO $b<a$ rồi mà  ~O)




#488646 $\boxed{\text{Chuyên Đề}}$ Bất đẳng thức - Cực trị

Đã gửi bởi lahantaithe99 on 24-03-2014 - 23:11 trong Bất đẳng thức và cực trị

$min của \sum \frac{x^{4}}{(x^{2}+y^{2})(x+y)} với x+y+z=3$ 

Áp dụng bđt Cô si

 

$\frac{x^4}{(x^2+y^2)(x+y)}+\frac{x^2+y^2}{4(x+y)}\geqslant \frac{x^2}{x+y}$

 

Thiết lập tương tự với các phân thức còn lại ta thu được

 

$\sum \frac{x^4}{(x^2+y^2)(x+y)}+\sum \frac{x^2+y^2}{4(x+y)}\geqslant \sum \frac{x^2}{x+y}$

 

$\sum \frac{x^4}{(x^2+y^2)(x+y)}\geqslant \sum \frac{x^2}{x+y}-\sum \frac{x^2+y^2}{4(x+y)}$

 

$=\frac{3}{4} \sum \frac{x^2}{x+y}-\sum \frac{y^2}{4(x+y)}=\frac{1}{2}\sum \frac{x^2}{x+y}+\frac{1}{4}\sum (\frac{x^2-y^2}{x+y})$

 

$=\frac{1}{2}\sum \frac{x^2}{x+y}+\frac{1}{4}\sum (x-y+y-z+z-x)=\frac{1}{2}\sum \frac{x^2}{x+y}$

 

Lịa có

 

$\frac{1}{2}\sum \frac{x^2}{x+y}\geqslant \frac{1}{2}.\frac{(x+y+z)^2}{2(x+y+z)}=\frac{3}{4}$

 

Do đó Min $\sum \frac{x^4}{(x^2+y^2)(x+y)}=\frac{3}{4}$




#488630 $\boxed{\text{Chuyên Đề}}$ Bất đẳng thức - Cực trị

Đã gửi bởi lahantaithe99 on 24-03-2014 - 22:06 trong Bất đẳng thức và cực trị

Bài 120:

Cho $x,y>0$ và $x+y=2$,. Tìm max $A=2xy(x^{2}+y^{2})$

 

Áp dụng bđt dạng $xy\leqslant \frac{(x+y)^2}{4}$ ta có

 

$2xy(x^2+y^2)\leqslant \frac{(2xy+x^2+y^2)^2}{4}=\frac{(x+y)^4}{4}=4$

 

P/s : bài nào xong thì nhớ tô đỏ theo quy định của box nhé  :icon6:




#488157 $\boxed{\text{Chuyên Đề}}$ Bất đẳng thức - Cực trị

Đã gửi bởi lahantaithe99 on 21-03-2014 - 21:24 trong Bất đẳng thức và cực trị

Mọi người giúp nhanh cho mình nha thanks nhìu  :(

114. Cho $x,y,z>0$ và $x+y+z=1$. Tính GTLN $P=\frac{1}{2x+y+z}+\frac{1}{x+2y+z}+\frac{1}{x+y+2z}$

 

115. Cho $x,y,z>0$ và $x+y+z=1$. Tính GTLN $s=\frac{x}{x+1}+\frac{y}{y+1}+\frac{z}{z+1}$

 

116. Cho $a,b,c>1$. CMR: $\frac{4a^{2}}{a-1}+\frac{5b^{2}}{b-1}+\frac{3c^{2}}{c-1}\geq 48$

 

117.Cho $a,b,c>0$. CMR $\frac{a^{3}}{b}+\frac{b^{3}}{c}+\frac{c^{3}}{a}\geq ab+bc+ca$

 

118. Cho $x,y>0$ và $x+y=1$. CMR $P=\frac{1}{x^{3}+y^{3}}+\frac{1}{xy}\geq 4+2\sqrt{3}$

Đã có tất trong đây

http://diendantoanho...1x2yzfrac1xy2z/




#489210 $\boxed{\text{Chuyên Đề}}$ Bất đẳng thức - Cực trị

Đã gửi bởi lahantaithe99 on 28-03-2014 - 17:30 trong Bất đẳng thức và cực trị

 

120) Cho $a;b;c>0$ thỏa $6(a^2+b^2+c^2)\geq 2011$. Tìm Min $A=\sum \frac{a^2}{b+c}$

 

 

 

Cũng là áp dụng BCS nhưng cách biến đổi có hơi khác chút

Ta có

 

$\sum \frac{a^2}{b+c}=\sum \frac{a^4}{a^2b+a^2c}\geqslant \frac{(a^2+b^2+c^2)^2}{ab(a+b)+bc(b+c)+ac(a+c)}$

 

Có BĐT  $ab(a+b)+bc(b+c)+ac(a+c)\leqslant \frac{2}{3}(a^2+b^2+c^2)(a+b+c)$

 

$\Rightarrow \sum \frac{a^2}{b+c}\geqslant \frac{3(a^2+b^2+c^2)}{2(a+b+c)}\geqslant \frac{3(a^2+b^2+c^2)}{2.\sqrt{3(a^2+b^2+c^2)}}$

 

$=\frac{1}{2}.\sqrt{3(a^2+b^2+c^2)}\geqslant \sqrt{\frac{2011}{8}}$